Đến nội dung

Hình ảnh

Topic tổng hợp các bài toán về phương trình nghiệm nguyên.


  • Please log in to reply
Chủ đề này có 565 trả lời

#261
Strygwyr

Strygwyr

    Sk8er-boi

  • Thành viên
  • 272 Bài viết

Bài 151 : Giải phương trình nghiệm nguyên dương :

$3^{x}+4^{y}=7^{z}$($1$)

Bài này nếu giải phương trình nghiệm nguyên thì khá là dài, tí nữa mình sẽ post sau.

Bài này giải như sau.

Nhận thấy $x=y=z=1$ là một nghiệm nguyên dương của phương trình ($1$). Ta sẽ chứng minh đây là nghiệm duy nhất của ($1$).

Thật vậy, xét $x\geq 2,y\geq 2,z\geq 2$

$\bullet$ Nếu $x=2k$ thì phương trình ($1$) tương đương với :

$3^{2k}+4^{y}=7^{z}\Leftrightarrow 9^{k}+4^{y}=7^{z}$

Vì $9\equiv 1(mod 8)$ và $4^{y}\equiv0(mod 8)$ với $y\geq 2$ nên 

$9^{k}+4^{y}\equiv 1 (mod 8)$ 

Mặt khác 

$7^{z}\equiv (-1)^{z}(mod 8$)

Suy ra $z$ chẵn. Đặt $z=2t$ với $t\in\mathbb{N}$, khi đó :

$3^{x}+4^{y}=7^{z}\Leftrightarrow 3^{x}=7^{2t}-2^{2y}=(7^{t}-2^{y})(7^{t}+2^{y})$

$\Leftrightarrow \left\{\begin{matrix}3^{u}=7^{t}+2^{y} & & \\ 3^{v}=7^{t}-2^{y}& & \end{matrix}\right.$

với $u,v\in\mathbb{N}$, $u+v=x$

Suy ra 

$3^{u}+3^{v}=2.7^{t}$

Lại do $7\equiv 1(mod 3)$ nên $2.7^{t}\equiv 2(mod 3)$

- Nếu $v=0$ thì $2.7^{t}=3^{u}+1\equiv 1(mod 3)$ (mâu thuẫn). Vậy phương trình vô nghiệm với $v=0$

- Nếu $v>0$ thì $2.7^{t}=3^{u}+3^{v}\equiv 0(mod 3)$ (mâu thuẫn). Vậy phương trình vô nghiệm với $v>0$.

Vậy với $x=2k$ thì phương trình đã cho vô nghiệm.

$\bullet$ Nếu $x=2k+1$ với $k\in\mathbb{N}$ thì phương trình ($1$) trở thành : 

$3^{2k+1}+4^{y}=7^{z}\Leftrightarrow 3.9^{k}+4^{y}=7^{z}$

Vì $9^{k}\equiv 1(mod 8)$ suy ra $3.9^{k}+4^{y}\equiv 3 (mod 8)$

Mặt khác $7^{z}\equiv (-1)^{z}=\pm 1(mod 8)$ suy ra mâu thuẫn..

Vậy với $x=2k+1$ thì phương trình đã cho vô nghiệm.

Vậy phương trình đã cho chỉ có một nghiệm nguyên dương duy nhất là $x=y=z=1$

Chơi luôn bài này 

Bài 152 : Tìm nghiệm nguyên của phương trình 

$3^{x}+4^{y}=7^{z}$


Bài viết đã được chỉnh sửa nội dung bởi namsub: 27-07-2013 - 15:46

"Nothing is impossible"

(Napoleon Bonaparte)


#262
Trang Luong

Trang Luong

    Đại úy

  • Thành viên
  • 1834 Bài viết

ờ công nhận trầm thật

ta có nếu x=0 hoặc y=0 thì (x,y)=(0,0) là 1 nghiệm của pt

ta xét x và y cùng khác 0 giả sử $\left | x \right |\geq \left | y \right |$

nếu trong 2 số x và y ko có số nào có giá trị tuyệt đối =1 thì $x^{2}+y^{2}\leq 2x^{2}\leq \frac{x^{2}y^{2}}{2}$

và $xy\leq x^{2}\leq \frac{x^{2}y^{2}}{4}$ do đó VT <VP ( vô lí)

do đó trong 2 số x,y tồn tại ít nhất 1 số có giá trị tuyệt đối =1

giả sử$\left | x \right |=1\Rightarrow 1\pm y+y^{2}=y^{2}+1$(vô lí vì y=0)

tương tự ta cũng có với $\left | y \right |=1$

vậy pt có nghiệm duy nhất (0,0)

Thiếu nghiệm rồi ạ (1,-1) và (-1,1)


"Nếu bạn hỏi một người giỏi trượt băng làm sao để thành công, anh ta sẽ nói với bạn: ngã, đứng dậy là thành công"
Issac Newton

#263
Trang Luong

Trang Luong

    Đại úy

  • Thành viên
  • 1834 Bài viết

Bài 135.

Cách khác : $x^{2}+y^{2}+xy=(xy)^{2}$

$\Rightarrow (x+y)^{2}=xy+(xy)^{2}=(xy+\frac{1}{2})^{2}-\frac{1}{4}$

$\Rightarrow (xy+\frac{1}{2})^{2}-(x+y)^{2}=\frac{1}{4}$

$\Leftrightarrow \left ( xy+\frac{1}{2}+x+y \right )\left ( xy+\frac{1}{2}-x-y \right )=\frac{1}{4}$

Vì $x,y$ nguyên nên $\Rightarrow \left ( xy+\frac{1}{2}+x+y \right )=\left ( xy+\frac{1}{2}-x-y \right )=\left | \frac{1}{2} \right |$

Nếu $\Rightarrow \left ( xy+\frac{1}{2}+x+y \right )=\left ( xy+\frac{1}{2}-x-y \right )=\frac{1}{2}\Rightarrow x=y=0$

Nếu $\Rightarrow \left ( xy+\frac{1}{2}+x+y \right )=\left ( xy+\frac{1}{2}-x-y \right )=\frac{-1}{2}\Rightarrow \left\{\begin{matrix} x+y=0 & & \\ xy=-1 & & \end{matrix}\right.$

...... Đến thế là xong 

P/s : Mọi người giải nốt bài 136,137 nhé  :icon6: 


"Nếu bạn hỏi một người giỏi trượt băng làm sao để thành công, anh ta sẽ nói với bạn: ngã, đứng dậy là thành công"
Issac Newton

#264
Trang Luong

Trang Luong

    Đại úy

  • Thành viên
  • 1834 Bài viết

Bài 152. GPT nghiệm nguyên :

$4y^{2}=2+\sqrt{199-x^{2}-2x}$


"Nếu bạn hỏi một người giỏi trượt băng làm sao để thành công, anh ta sẽ nói với bạn: ngã, đứng dậy là thành công"
Issac Newton

#265
Juliel

Juliel

    Thượng úy

  • Thành viên
  • 1240 Bài viết

Bài này nếu giải phương trình nghiệm nguyên thì khá là dài, tí nữa mình sẽ post sau.

Bài này giải như sau.

Nhận thấy $x=y=z=1$ là một nghiệm nguyên dương của phương trình ($1$). Ta sẽ chứng minh đây là nghiệm duy nhất của ($1$).

Thật vậy, xét $x\geq 2,y\geq 2,z\geq 2$

$\bullet$ Nếu $x=2k$ thì phương trình ($1$) tương đương với :

$3^{2k}+4^{y}=7^{z}\Leftrightarrow 9^{k}+4^{y}=7^{z}$

Vì $9\equiv 1(mod 8)$ và $4^{y}\equiv0(mod 8)$ với $y\geq 2$ nên 

$9^{k}+4^{y}\equiv 1 (mod 8)$ 

Mặt khác 

$7^{z}\equiv (-1)^{z}(mod 8$)

Suy ra $z$ chẵn. Đặt $z=2t$ với $t\in\mathbb{N}$, khi đó :

$3^{x}+4^{y}=7^{z}\Leftrightarrow 3^{x}=7^{2t}-2^{2y}=(7^{t}-2^{y})(7^{t}+2^{y})$

$\Leftrightarrow \left\{\begin{matrix}3^{u}=7^{t}+2^{y} & & \\ 3^{v}=7^{t}-2^{y}& & \end{matrix}\right.$

với $u,v\in\mathbb{N}$, $u+v=x$

Suy ra 

$3^{u}+3^{v}=2.7^{t}$

Lại do $7\equiv 1(mod 3)$ nên $2.7^{t}\equiv 2(mod 3)$

- Nếu $v=0$ thì $2.7^{t}=3^{u}+1\equiv 1(mod 3)$ (mâu thuẫn). Vậy phương trình vô nghiệm với $v=0$

- Nếu $v>0$ thì $2.7^{t}=3^{u}+3^{v}\equiv 0(mod 3)$ (mâu thuẫn). Vậy phương trình vô nghiệm với $v>0$.

Vậy với $x=2k$ thì phương trình đã cho vô nghiệm.

$\bullet$ Nếu $x=2k+1$ với $k\in\mathbb{N}$ thì phương trình ($1$) trở thành : 

$3^{2k+1}+4^{y}=7^{z}\Leftrightarrow 3.9^{k}+4^{y}=7^{z}$

Vì $9^{k}\equiv 1(mod 8)$ suy ra $3.9^{k}+4^{y}\equiv 3 (mod 8)$

Mặt khác $7^{z}\equiv (-1)^{z}=\pm 1(mod 8)$ suy ra mâu thuẫn..

Vậy với $x=2k+1$ thì phương trình đã cho vô nghiệm.

Vậy phương trình đã cho chỉ có một nghiệm nguyên dương duy nhất là $x=y=z=1$

Bác giải thiếu rồi đấy, bác chỉ ra một nghiệm $x=y=z=1$ và xét trường hợp $x,y,z\geq 2$

Lỡ đâu một trong ba số $x,y,z$ bằng $1$ và hai số còn lại lớn hơn hoặc bằng $2$ thì sao ?

Bài này khó ở chỗ xét $y=1;x,z\geq 2$, phần còn lại thì khá đơn giản, vậy mà bác đã bỏ qua trường hợp này  :angry:


Bài viết đã được chỉnh sửa nội dung bởi Juliel: 27-07-2013 - 18:25

Đừng rời xa tôi vì tôi lỡ yêu người mất rồi !
 

Welcome to My Facebook !


#266
Ha Manh Huu

Ha Manh Huu

    Trung úy

  • Thành viên
  • 799 Bài viết

Bài 152. GPT nghiệm nguyên :

$4y^{2}=2+\sqrt{199-x^{2}-2x}$

có $4y^{2} <2+15 $ do đó ok


tàn lụi


#267
DarkBlood

DarkBlood

    Thiếu úy

  • Thành viên
  • 619 Bài viết

Ủng hộ topic phát (bài nào trùng nhờ Jinbe xoá giùm :)))

Bài 142 : Giải phương trình nghiệm nguyên tố $p$,$q$,$r$ : 

$p(p+1)+q(q+1)=r(r+1)$

 

Bài 145 (Bài toán $Lebesgue$) Chứng minh rằng phương trình $x^{2}-y^{3}=7$ không có nghiệm nguyên.

Bài 142: http://diendantoanho...yen-thcs/page-3

Tại #60 :)

 

Bài 145: Áp dụng hai bổ đề:

Bổ đề 1: Mỗi số nguyên có dạng $4a+3$ $(a\in \mathbb{N})$ thì tồn tại ít nhất một ước nguyên tố có dạng $4b+3$ $(b\in \mathbb{N}).$

Bổ đề 2: Nếu $x,\ y$ nguyên, $p$ là số nguyên tố có dạng $4k+3$ $(k\in \mathbb{N})$ thỏa mãn $p\mid x^2+y^2$ thì $p\mid x$ và $p\mid y.$

 

Lời giải

Ta có phương trình đã cho tương đương với $x^2+1=y^3+8\ \ \ (1)$

Xét hai trường hợp:

Trường hợp 1: $y$ chẵn. Từ $(1)$ suy ra $x$ lẻ.

Khi đó $VT\equiv 2\ (\bmod\ 4)\ ;\ VP\equiv 0\ (\bmod\ 4)$ $($Vô lý$)$

 

Trường hợp 2: $y$ lẻ. Ta có:

$(1)\ \Leftrightarrow\ x^2+1=(y+2)(y^2-2y+4)=(y+2)[(y-1)^2+3]$

Vì $y$ lẻ nên $4\mid (y-1)^2,$ suy ra $(y-1)^2+3=4a+3$ $(a\in \mathbb{N}).$ Theo bổ đề 1 thì $(y-1)^2+3$ có ước nguyên tố dạng $p=4b+3$ $(b\in \mathbb{N}).$

Do đó $x^2+1$ có ước nguyên tố dạng $p=4b+3$ $(b\in \mathbb{N}).$

Áp dụng bổ đề 2, ta có: $p\mid 1$ $($Vô lý$)$

 

Vậy phương trình vô nghiệm.



#268
Strygwyr

Strygwyr

    Sk8er-boi

  • Thành viên
  • 272 Bài viết

tôi ko biết làm có đúng ko nhưng cứ viết vậy

 

do$x^{2}\equiv 0,1(mod;4)\Rightarrow x^{2}+y^{2}\not\equiv 3 mod 4$ (vô lí)

 

 

Chém luôn, chả biết đúng hay sai.

Bổ đề quen thuộc : Nếu $x,y$ nguyên và $p$ nguyên tố có dạng $4k + 3$ thỏa $p|x^{2}+y^{2}$ thì $p|x$ và $p|y$

Giải :

Gỉa sử trong phân tích tiêu chuẩn của $n$, các ước nguyên tố $p_{1},p_{2},...,p_{i}$ có dạng $4k+3$ đều có số mũ lẻ

Khi đó, đặt $n=p_{1}^{k_{1}}.p_{2}^{k_{2}}...p_{i}^{k_{i}}.q$ với $k_{1},k_{2},...,k_{i}$ đều lẻ và $q$ nguyên tố cùng nhau với tất cả các số nguyên tố $p_{1},p_{2},...,p_{i}$

Ta có quyền giả sử $k_{1}=min\left \{ k_{1};k_{2};...;k_{i} \right \}$

Do đó từ bổ đề trên, ta suy ra :

$x$ và $y$ đều chia hết cho tất cả các số nguyên tố $p_{1},p_{2},...,p_{i}$

Đặt $x=p_{1}p_{2}...p_{i}.x_{1}$ và $y=p_{1}p_{2}...p_{i}.y_{1}$

Thay vào phương trình và thu gọn, ta được :

$$x_{1}^{2}+y_{1}^{2}=p_{1}^{k_{1}-2}.p_{2}^{k_{2}-2}....p_{i}^{k_{i}-2}.q$$

Do $k_{1},k_{2},...,k_{i}$ đều lẻ và $k_{1}$ nhỏ nhất nên cứ tiếp tục như vậy, cho đến một lúc nào đó thì ta có :

$$x_{m}^{2}+y_{m}^{2}=p_{1}.p_{2}^{k_{2}-2t}....p_{i}^{k_{i}-2t}.q\Rightarrow \left\{\begin{matrix} p_{1}|x_{m} & & \\ p_{1}|y_{m}& & \end{matrix}\right.\Rightarrow p_{1}^{2}|x_{m}^{2}+y_{m}^{2}\Rightarrow p_{1}^{2}|p_{1}.p_{2}^{k_{2}-2t}...p_{i}^{k_{i}-2t}$$

Điều này hiển nhiên vô lí

Vậy : Trong phân tích tiêu chuẩn của $n$ thì các thừa số nguyên tố có dạng $4k + 3$ phải có số mũ chẵn

 

Về hai bài giải này mình chưa đề cập về kiến thức và trước hết là về kĩ năng biên luận phương trình $Diophant$ bậc $2$. Các bạn nên lưu ý đối với các bài toán có dạng chứng minh $A$ khi và chỉ khi thoả mãn $B$ ta nên trình bày dưới $2$ phần :

 

$\bullet$ Điều kiện cần : Để có $A$ thì phải thoả mãn $B$.

$\bullet$ Điều kiện đủ : Nếu thoả mãn $B$ thì có $A$.

 

Chẳng hạn như bài 146 (bài dễ nhất) ta có thể giải như sau :

Bài 146 : Cho số nguyên tố $p$. Chứng minh rằng phương trình :

$x^{2}+y^{2}=p$

có nghiệm nguyên dương khi và chỉ khi $p$ không có dạng $4k+3$

$\bullet$ Điều kiện cần : Giả sử phương trình $x^{2}+y^{2}=p$ có nghiệm nguyên dương.

Dễ thấy $x^{2}\equiv 0;1(mod 4)$ nên $p=x^{2}+y^{2}\equiv 0;1;2 (mod 4)$

Suy ra $p$ không thể có dạng $4k+3$.

$\bullet$ Điều kiện đủ : Giả sử $p$ không có dạng $4k+3$. Lúc đó $p=2$ hoặc $p=4k+1$

- Nếu $p=2$ thì phương trình đã cho có nghiệm nguyên dương $x=y=1$

- Nếu $p=4k+1$ thì ta có bài toán quen thuộc :

Bài toán : (Định lí $Fermat-Euler$) Chứng minh rằng với mọi số nguyên tố $p$ có dạng $4k+1$ thì $p$ luôn biểu diễn được dưới dạng tổng hai bình phương của $2$ số nguyên dương.

Giả sử $p=4k+1$

Xét $a=(2k)!=(-1)^{2k}(2k)!=(-1)(-2)...(-2k)\equiv (p-1)(p-2)...(p-2k)=(4k)(4k-1)...(2k+1)(mod p)$

Vậy $a^{2}\equiv (2k)!(4k)(4k-1)...(2k+1)=(4k)!=(p-1)!(mod p)$

Theo định lý $Wilson$ thì $(p-1)!\equiv -1(mod p)$

Suy ra $a^{2}\equiv -1(mod p)$

Kí hiệu $q=[\sqrt{p}]$. Xét $(q+1)^{2}$ số có dạng $ax+y$ với 

$x=0,1,...,q$

 

$y=0,1,...,q$

 

Vì $(q+1)^{2}>p>q^{2}$ nên tồn tại cặp ($x_1,y_1$) và ($x_2,y_2$) sao cho $ax_1+y_1\equiv ax_2+y_2(mod p)$

hay $p|a(x_1-x_2)+(y_1+y_2)$

Đặt $x=\left | x_1-x_2 \right |$ và $y=\left | y_1-y_2 \right |$

Ta có : $a^{2}x^{2}-y^{2}=(ax-y)(ax+y)\vdots p$

Lại có $a^{2}\equiv -1(mod p)$ nên $x^{2}+y^{2}\equiv -a^{2}x^{2}+y^{2}\equiv 0 (mod p)$

Do $x^{2}\leq q^{2}\leq p$ và $y^{2}\leq q^{2}\leq p$, với chú ý $p$ nguyên tố ta suy ra được $x^{2}+y^{2}<2p$

Suy ra $x^{2}+y^{2}=p$. Dễ thấy $x\neq 0$, $y\neq 0$

Chứng minh hoàn tất.


"Nothing is impossible"

(Napoleon Bonaparte)


#269
Strygwyr

Strygwyr

    Sk8er-boi

  • Thành viên
  • 272 Bài viết

Thể theo nguyện vọng của Juliel, mình xin phép xử đẹp bài $148$ :P

Bài 148 : Cho số nguyên dương $n$ không phải là số chính phương. Chứng minh phương trình 

$x^{2}+y^{2}=n$ ($1$)

có nghiệm nguyên dương khi và chỉ khi trong phân tích tiêu chuẩn của $n$ các số nguyên tố dạng $4k+3$ có số mũ chẵn.

$\bullet$ Điều kiện cần : 

Trước khi chứng minh, ta có một bổ đề quen thuộc :

Bổ đề 1 : Cho số nguyên tố $p$ có dạng $4k+3$ và các số nguyên dương $x$,$y$ sao cho $p|x^{2}+y^{2}$. Khi đó $p|x$ và $p|y$.

 

Trở lại bài toán, gọi $m^{2}$ là ước chính phương lớn nhất của $n$. Khi đó $n=m^{2}.l$ với $l$ không phải là số chính phương.

Ta cần chứng minh mọi ước của $l$ đều có dạng $4k+1$.

Giả sử $x$, $y$ là một nghiệm của ($1$). Đặt $d=(x,y), x=x_1d, y=y_1d$ với $(x_1,y_1)=1$

Ta có : $d^{2}(x_1^{2}+y_1^{2})=m^{2}.l$

Vì $m^{2}$ là ước chính phương lớn nhất của $n$, và $d=(x,y)$ với $x^{2}+y^{2}=n$ nên suy ra $d^{2}|m^{2}\Rightarrow d|m$ 

Đặt $m=dm_1$ thì ta có : $x_1^{2}+y_1^{2}=m_1^{2}l$

Giả sử $p$ có một ước nguyên tố $q$ nào đó có dạng $4k+3$ thì theo bổ đề $1$ thì $q|x_1,q|y_1$ (mâu thuẫn với $(x_1,y_1)=1$)

Điều kiện cần chứng minh xong.

 

$\bullet$ Điều kiện đủ : 

Chứng minh điều kiện đủ ta cần có $2$ bổ đề 

Bổ đề 2 (Định lí $Fermat-Euler$) : Mọi số nguyên tố $p$ có dạng $4k+1$ đều có thể viết được dưới dạng tổng bình phương của hai số nguyên dương (xem chứng minh ở trên)

Bổ đề 3 Tích của hai số lẻ, mỗi số lẻ là tổng bình phương của hai số nguyên dương cũng sẽ là tổng bình phương của hai số nguyên dương.

Chứng minh : Giả sử 

$n=a^{2}+b^{2}$ và $m=c^{2}+d^{2}$

Khi đó : 

$mn=(a^{2}+b^{2})(c^{2}+d^{2})=(ad+bc)^{2}+(ac-bd)^{2}=(ac+bd)^{2}+(ad-bc)^{2}$

Nếu $ac=bd$ và $ab=cd$ thì $a=b$ và $c=d$. Khi đó $m$ và $n$ là các số chẵn, mâu thuẫn.

Nếu ít nhất một trong hai số $ac-bd$ và $ab-cd$ khác $0$ thì lúc đó $mn$ là tổng bình phương của hai số nguyên dương.

 

Trở lại bài toán, đặt $n=m^{2}l$ với $l=2^{\alpha }p_1p_2...p_n$ với $\alpha$ bằng $0$ hoặc $1$ còn $p_i$ ($i=1,2,...,n$) là các số nguyên tố dạng $4k+1$.

Theo bổ đề $2$ và $3$, tồn tại các số nguyên dương $a$ và $b$ sao cho $p_1p_2...p_n=a^{2}+b^{2}$

Suy ra $(ma)^{2}+(mb)^{2}=m^{2}p_1p_2...p_n$

- Nếu $\alpha =0$ thì bài toán chứng minh xong.

- Nếu $\alpha =1$ thì ta có : $n=2m^{2}p_1p_2...p_n=2((ma)^{2}+(mb)^{2})=(ma+mb)^{2}+(ma-mb)^{2}$

Do $a\neq b$ nên bài toán chứng minh xong.


Bài viết đã được chỉnh sửa nội dung bởi namsub: 28-07-2013 - 17:37

"Nothing is impossible"

(Napoleon Bonaparte)


#270
Zaraki

Zaraki

    PQT

  • Phó Quản lý Toán Cao cấp
  • 4273 Bài viết

Anh namsub cho em hỏi tí, chỗ anh đặt $l=2^{ \alpha} p_1p_1 \cdots p_n$ sao $\alpha$ chỉ có thể là $0$ hoặc $1$, em nghĩ là $2,3...$ vẫn được chớ.


Discovery is a child’s privilege. I mean the small child, the child who is not afraid to be wrong, to look silly, to not be serious, and to act differently from everyone else. He is also not afraid that the things he is interested in are in bad taste or turn out to be different from his expectations, from what they should be, or rather he is not afraid of what they actually are. He ignores the silent and flawless consensus that is part of the air we breathe – the consensus of all the people who are, or are reputed to be, reasonable.

 

Grothendieck, Récoltes et Semailles (“Crops and Seeds”). 


#271
Juliel

Juliel

    Thượng úy

  • Thành viên
  • 1240 Bài viết

Bài 152 : Tìm nghiệm nguyên của phương trình 

$3^{x}+4^{y}=7^{z}$

Xét hơi nhiều trường hợp tý nhé !

$\boxed{1}$ Xét trường hợp $x < 0$ :

$\bullet$ $y > 0$ và $z > 0$ : Một vế nguyên, một vế không nguyên (loại)

$\bullet$ $y > 0$ và $z < 0$ : 

$\frac{1}{3^{|x|}}+4^{y}=\frac{1}{7^{|z|}}\Leftrightarrow 7^{|z|}+4^{y}.3^{|x|}.7^{|z|}=3^{|x|}$

Vô lí vì hiển nhiên $7^{|z|}+4^{y}.3^{|x|}.7^{|z|}>3^{|x|}$

$\bullet$ $y < 0$ và $z > 0$ :

$\frac{1}{3^{|x|}}+\frac{1}{4^{|y|}}=7^{z}\Leftrightarrow 3^{|x|}+4^{|y|}=7^{z}.3^{|x|}.4^{|y|}$

Một vế chẵn, một vế lẻ (loại)

$\bullet$ $y < 0$ và $z < 0$ :

$\frac{1}{3^{|x|}}+\frac{1}{4^{|y|}}=\frac{1}{7^{|z|}}\Leftrightarrow 7^{|z|}.(3^{|x|}+4^{|y|})=3^{|x|}.4^{|y|}$

Một vế chẵn, một vế lẻ (loại)

 

$\boxed{2}$ Xét trường hợp : $x\geq 0$ 

$\bullet$ $y\geq 0;z\geq 0$  (sẽ giải sau)

$\bullet$ $y\geq 0;z\leq 0$ : Một vế nguyên, một vế không nguyên (loại)

$\bullet$ $y\leq 0;z\geq 0$ : Một vế nguyên, một vế không nguyên (loại)

$\bullet$ $y\leq 0;z\leq 0$ : $3^{|x|}+\frac{1}{4^{|y|}}=\frac{1}{7^{|z|}}\Leftrightarrow 3^{|x|}.7^{|z|}.4^{|y|}+7^{|z|}=4^{|y|}$

Vô lí vì hiển nhiên $3^{|x|}.7^{|z|}.4^{|y|}+7^{|z|}>4^{|y|}$

 

Bây giờ ta sẽ giải trường hợp cả ẩn $x,y,z$ đều là những số nguyên dương

$\blacksquare$ Nếu $y\geq 2$ thì : $4^{y}\vdots 8$

Ta có : $3^{x}+4^{y}\equiv 1;3(mod8)$ mà $7^{z}\equiv 1;7(mod8)$

Do đó $7^{z}=3^{x}+4^{y}\equiv 1(mod8)\Rightarrow x,z$ chẵn

Đặt $x=2a;y=2b$ (với $a,b$ nguyên dương), ta được :

$$3^{2a}+2^{2y}=7^{2b}\Leftrightarrow (7^{b}-3^{a})(7^{b}-3^{a})=2^{2y}$$

Do đó :

$$\left\{\begin{matrix} 7^{b}-3^{a}=2^{m} & & \\ 7^{b}+3^{a}=2^{n} & & \end{matrix}\right.$$

Với $m,n$ tự nhiên và $m<n,m + n = 2y$

Trừ vế với vế :

$$2.3^{a}=2^{n}-2^{m}$$

$\Leftrightarrow 2.3^{a}=2^{m}(2^{n-m}-1)\Rightarrow \left\{\begin{matrix} m=1 & & \\ 2^{n-1}-1=3^{a}& & \end{matrix}\right.$

Xét riêng phương trình $2^{n-1}-1=3^{a}$. Bằng cách xét đồng dư mô-đun $3$, chỉ ra được $n-1$ chẵn.

Từ đó dễ tìm được $a=1;n=3;m=1$. Từ đó $y = 1$ (loại vì đang xét $y\geq 2$)

$\blacksquare$ Ta xét $y = 1$

Ta được phương trình : $$3^{x}+4=7^{z}$$

Dễ thấy khi $z = 1$ thì $x = 1$, ta xét $z > 1$

$\blacktriangledown$ Xét $z$ chẵn, thì $7^{z}\equiv 1(mod4)\Rightarrow 3^{x}+4\equiv 1(mod4)\Rightarrow x$ chẵn

Đặt $z=2m;y=2n$ ($m,n$ nguyên dương), thay vào thì : $(7^{m}-3^{n})(7^{m}+3^{n})=4$

Phương trình ước số này không cho nghiệm thỏa mãn

$\blacktriangledown$ Xét $z$ lẻ thì : $7^{z}\equiv 3(mod4)\Rightarrow 3^{x}+4\equiv 3(mod4)\Rightarrow x$ lẻ

Bằng cách đặt $x = 6k + r$ và xét đồng dư theo mô-đun $7$ ta dễ dàng tìm được $r=1$, tức $x = 6k+1$

Vì $x$ lẻ nên $$3^{x}\equiv 2;3(mod5)\Rightarrow 7^{z}=3^{x}+4\equiv 1;2(mod5)(\bigstar )$$

Vì $z$ lẻ nên đặt $z = 2z_{1}+1$, ta được : $$7^{z}=7^{2z_{1}+1}\equiv (-1)^{z_{1}}.7\equiv 2;3(mod5)(\blacklozenge )$$

Từ $(\blacklozenge )(\bigstar )$ suy ra :

$$(-1)^{z_{1}}.7\equiv 2(mod5)\Rightarrow z_{1}\vdots 2\Rightarrow z\equiv 1(mod4)$$

Ta có $y = 6k+1$ nên : 

$$7^{z}=3^{x}+4=3^{6k+1}+4\equiv 7(mod13)$$.

Đặt $z = 12t + r'$ thì $$7^{z}=7^{12t+r'}\equiv 7^{r'}\equiv 7(mod13)\Rightarrow r'=1\Rightarrow z=12t+1$$

 

Như vậy ta đã chứng minh xong $z = 12t+1$ và $x = 6k+1$

Từ phương trình đã cho, suy ra :

$$7^{z}-7=3^{x}-3\Leftrightarrow 7^{12t+1}-7=3^{6k+1}-3\Leftrightarrow 7(7^{12t}-1)=3(3^{6k}-1)$$

Ta thấy $VT\equiv 0(mod3);VP\equiv 6(mod9)$ (loại)

 

KẾT LUẬN : $\boxed{(x;y;z)=(1;1;1)}$


Bài viết đã được chỉnh sửa nội dung bởi Juliel: 20-08-2013 - 19:02

Đừng rời xa tôi vì tôi lỡ yêu người mất rồi !
 

Welcome to My Facebook !


#272
Strygwyr

Strygwyr

    Sk8er-boi

  • Thành viên
  • 272 Bài viết

Anh namsub cho em hỏi tí, chỗ anh đặt $l=2^{ \alpha} p_1p_1 \cdots p_n$ sao $\alpha$ chỉ có thể là $0$ hoặc $1$, em nghĩ là $2,3...$ vẫn được chớ.

Thế này Toàn nè, chỗ ấy anh đặt $n=m^{2}l$ với $l=2^{\alpha }p_1p_2...p_n$ với $\alpha$ bằng $0$ hoặc $1$. Nên nếu $\alpha$ lớn hơn 1 thì nó chuyển sang bên $m^{2}$ nữa cơ mà.

À mình có ý này, để topic tiếp tục phát triển và chất lượng cao hơn, hay là ta dịch mấy bài bên đề MO và TST của các nước về phương trình nghiệm nguyên vào nhỉ :)

Spoiler


"Nothing is impossible"

(Napoleon Bonaparte)


#273
Juliel

Juliel

    Thượng úy

  • Thành viên
  • 1240 Bài viết

(còn nhiều bài chưa giải quá nhỉ)

Bài 153 : Giải phương trình nghiệm nguyên :

$$4^{x}+4^{y}+2^{xy}-2^{x^{2}}-2^{y^{2}}=16$$


Đừng rời xa tôi vì tôi lỡ yêu người mất rồi !
 

Welcome to My Facebook !


#274
Strygwyr

Strygwyr

    Sk8er-boi

  • Thành viên
  • 272 Bài viết

Bài 147 : Cho số chính phương $n$ khác $0$. Chứng minh rằng phương trình

$x^{2}+y^{2}=n$

có nghiệm nguyên dương khi và chỉ khi $n$ có ít nhất một ước nguyên tố có dạng $4k+1$.

Topic giờ đóng bụi ghê, xử đẹp bài này của mình trước vậy :P

Điều kiện cần :

Giả sử $n=m^{2}$. Gọi $x$,$y$ là hai nghiệm của phương trình đã cho. Đặt $d=(x,y)$ và $x=dx_1,y=dy_1$ với $(x_1,y_1)=1$

Suy ra $d^{2}(x_1^{2}+y_1^{2})=m^{2}\Rightarrow m=dm_1\Rightarrow x_1^{2}+y_1^{2}=m_1^{2}$

Nhận thấy $m_1$ lẻ vì nếu $m_1$ chắn thì $4|x_1^{2}+y_1^{2}$ suy ra $x_1$ và $y_1$ cùng chẵn (mâu thuẫn với $(x_1,y_1)=1$)

Gọi $p$ là một ước nguyên tố của $m_1$. Dễ thấy $(x_1,p)=(y_1,p)=1$

Theo định lý $Fermat$ nhỏ thì

$x_1^{p-1}\equiv 1 (mod p)$ và $y_1^{p-1}\equiv 1 (mod p)$

Do $p|x_1^{2}+y_1^{2}\Rightarrow x_1^{2}\equiv (-1)y_1^{2}(mod p)\Rightarrow x_1^{p-1}\equiv (-1)^{\frac{p-1}{2}}y_1^{p-1}$

SUy ra $\frac{p-1}{2}$ chắn nên $p$ có dạng $4k+1$

Mọi ước nguyên tố của $m_1$ đều có dạng $4k+1$ nên $m$ có ước dạng $4k+1$

Điều kiện đủ :

Bổ đề 1 : (định lý $Fermat-Euler$) Mọi số nguyên tố có dạng $4k+1$ đều có thể biểu diễn dưới dạng tổng bình phương của hai số nguyên dương.

Bổ đề 2 : Tích của hai số lẻ, mỗi số là tổng bình phương của hai số chính phương thì có thể biểu diễn được dưới dạng tổng bình phương của hai số chính phương.

Chứng minh hai bổ đề này ở trên.

Trở lại bài toán, đặt $n=m^{2}$ với $m=m_1p$ với $p$ là số nguyên tố dạng $4k+1$. Theo bổ đề $1$ và $2$, tồn tại hai sô nguyên dương $a$ và $b$ sao cho : 

$a^{2}+b^{2}=p^{2}$

Vậy $(m_1a)^{2}+(m_1b)^{2}=(m_1p)^{2}=m^{2}=n$


Bài viết đã được chỉnh sửa nội dung bởi Strygwyr: 03-08-2013 - 12:26

"Nothing is impossible"

(Napoleon Bonaparte)


#275
tieutuhamchoi98

tieutuhamchoi98

    Trung sĩ

  • Banned
  • 173 Bài viết

Có được đóng góp thêm cho sôi nổi lại cái topic này k nhỉ? 



#276
Strygwyr

Strygwyr

    Sk8er-boi

  • Thành viên
  • 272 Bài viết

Ủng hộ topic một số bài trong Junior Balkan Mathematical Olympiad nào :))

Bài 154 : Tìm các số nguyên dương $x$, $y$ thỏa mãn :

$x^{y}=y^{x-y}$

(Junior Balkan 1998)

Bài 155 : Tìm các số nguyên dương $x$ sao cho $n^{2}+3^{n}$ là số chính phương.

(Junior Balkan 2000)

Bài 156 : Tìm nghiệm nguyên dương của phương trình : 

$a^{3}+b^{3}+c^{3}=2001$

(Junior Balkan 2001)

Bài 157 : Tìm nghiệm nguyên dương của phương trình :

$9(x^{2}+y^{2}+1)+2(3xy+2)=2005$

(Junior Balkan 2005)

Bài 158 : Tìm nghiệm nguyên tố $p,q,r$ của phương trình sau :

$\frac{p}{q}-\frac{4}{r+1}=1$

(Junior Balkan 2008)

Bài 159 : Tìm nghiệm nguyên không âm của phương trình sau :

$2^{a}3^{b}+9=c^{2}$

(Junior Balkan 2009)

Bài 160 : Tìm các số nguyên dương $n$ sao cho $n.2^{n+1}+1$ là số chính phương.

(Junior Balkan 2010)

Bài 161 : Tìm các số nguyên tố $p$ sao cho tồn tại các số nguyên dương $x$ và $y$ thỏa mãn :

$x(y^{2}-p)+y(x^{2}-p)=5p$

(Junior Balkan 2011)

Bài 162 : Tìm các số nguyên dương $x,y,z,t$ sao cho 

$2^{x}3^{y}+5^{z}=7^{t}$

(Junior Balkan 2012)

@tieutuhamchoi98 : bạn có thể post đề thoải mái nhưng hiện tại do mình đã post khá nhiều và cũng có nhiều bài chưa có lời giải nên bạn nên giải những bài toán này thì hơn :))


Bài viết đã được chỉnh sửa nội dung bởi Strygwyr: 04-08-2013 - 10:23

"Nothing is impossible"

(Napoleon Bonaparte)


#277
bangbang1412

bangbang1412

    Độc cô cầu bại

  • Phó Quản lý Toán Cao cấp
  • 1670 Bài viết

có một cách khác ; thật vậy phương trình tương đương là $(x+y)^{2}=xy(xy+1)$ ; vế trái là một số chính phương ; vế phải là tích 2 số nguyên liên tiếp nên 1 trong 2 số xy và xy + 1 là 0 ; dễ thấy từ điều kiện ban đầu thì xy  >= 0 nên xy = 0 ; do đó x =y = 0 là nghiệm duy nhất .

ờ công nhận trầm thật

ta có nếu x=0 hoặc y=0 thì (x,y)=(0,0) là 1 nghiệm của pt

ta xét x và y cùng khác 0 giả sử $\left | x \right |\geq \left | y \right |$

nếu trong 2 số x và y ko có số nào có giá trị tuyệt đối =1 thì $x^{2}+y^{2}\leq 2x^{2}\leq \frac{x^{2}y^{2}}{2}$

và $xy\leq x^{2}\leq \frac{x^{2}y^{2}}{4}$ do đó VT <VP ( vô lí)

do đó trong 2 số x,y tồn tại ít nhất 1 số có giá trị tuyệt đối =1

giả sử$\left | x \right |=1\Rightarrow 1\pm y+y^{2}=y^{2}+1$(vô lí vì y=0)

tương tự ta cũng có với $\left | y \right |=1$

vậy pt có nghiệm duy nhất (0,0)


$$[\Psi_f(\mathbb{1}_{X_{\eta}}) ] = \sum_{\varnothing \neq J} (-1)^{\left|J \right|-1} [\mathrm{M}_{X_{\sigma},c}^{\vee}(\widetilde{D}_J^{\circ} \times_k \mathbf{G}_{m,k}^{\left|J \right|-1})] \in K_0(\mathbf{SH}_{\mathfrak{M},ct}(X_{\sigma})).$$


#278
Juliel

Juliel

    Thượng úy

  • Thành viên
  • 1240 Bài viết


 

Bài 159 : Tìm nghiệm nguyên không âm của phương trình sau :

$2^{a}3^{b}+9=c^{2}$

(Junior Balkan 2009)

 

Phương trình tương đương :

$$(c-3)(c+3)=2^{a}.3^{b}$$

Khi $a = 0$ thì $b = 3$, $c = 6$ 

Khi $b = 0$ thì $a = 4$, tìm được $c = 5$

Khi $a  = b = 0$ thì $c$ tìm được không nguyên

Xét $a,b>0$ :

Dễ thấy $c - 3$ và $c + 3$ phải cùng là số chẵn

Xét các trường hợp :

  • $\left\{\begin{matrix} c-3 =2^{m}.3^{b}& & \\ c+3=2^{n}& & \end{matrix}\right.$

Trừ vế : $$2^{n}-2^{m}.3^{b}=6$$

Vì $b >0$ nên $VT$ không chia hết cho $3$, $VP$ chia hết cho $3$ (loại)

  • $\left\{\begin{matrix} c-3=2^{m} & & \\ c+3=2^{n}.3^{b}& & \end{matrix}\right.$

Trường hợp này tương tự như trên

  • $\left\{\begin{matrix} c-3=2^{m}.3^{x} & & \\ c+3=2^{n}.3^{y}& & \end{matrix}\right.$

Trừ vế : $$2^{n}.3^{y}-2^{m}.3^{x}=6$$

(+) Nếu $m\geq n,x\geq y\Rightarrow 6=2^{n}.3^{y}-2^{m}.3^{x}\leq 0$ (vô lí)

(+) Nếu $m\leq n,x\geq y$ thì : $$2^{m}.3^{y}(2^{n-m}-3^{x-y})=2.3\Rightarrow \left\{\begin{matrix} m=y=1 & & \\ 2^{n-1}-3^{x-1}=1& & \end{matrix}\right.$$

Xét riêng phương trình $2^{n-1}=1+3^{x-1}$

Bằng cách xét đồng dư theo mô-đun $3$, ta chỉ ra được $n - 1$ chẵn, từ đó đặt $n - 1 = 2k$

$$2^{2k}=1+3^{x-1}\Leftrightarrow (2^{k}-1)(2^{k}+1)=3^{x-1}\Rightarrow k=1;x=2\Rightarrow n=3;x=2$$

Suy ra $a = 4 ; b = 3$, suy ra $c = 21$

(+) Nếu $m\geq n;x\leq y$

Khi đó : $$2^{n}.3^{x}(3^{y-x}-2^{m-n})=2.3\Rightarrow \left\{\begin{matrix} n=x=1 & & \\ 3^{y-1}-2^{m-1}=1 & & \end{matrix}\right.$$

Xét riêng phương trình $3^{y-1}=2^{m-1}+1$

Phương trình này cho nghiệm $m = y = 2$, suy ra $a = b = 3, c = 15$

Xét $m>2\Rightarrow 4|2^{m-1}$, bằng cách xét đồng dư mô-đun $4$, lại suy được $y -1$ chẵn, đặt $y- 1=2k$

Ta được $$3^{2k}=2^{m-1}+1\Leftrightarrow (3^{k}-1)(3^{k}+1)=2^{m-1}\Rightarrow k=1;m=4\Rightarrow y=3;m=4$$

Suy ra $a = 5;b = 4$, được $c = 51$

(+) Nếu $m\leq n;x\leq y$ thì :

$$2^{m}.3^{x}(2^{n-m}.3^{y-x}-1)=2.3\Rightarrow \left\{\begin{matrix} m=x=1 & & \\ 2^{n-1}.3^{y-1}-1=1 & & \end{matrix}\right.$$

Xét riêng phương trình $2^{n-1}.3^{y-1}=2\Rightarrow n=2;y=1$

Từ đó $a = 3;b = 2$ được $c = 9$

Kết luận : $\boxed{(a;b;c)=(4;0;5);(4;3;21);(5;4;51);(3;2;9);(3;3;15);(0;3;6)}$


Bài viết đã được chỉnh sửa nội dung bởi Juliel: 20-08-2013 - 18:54

Đừng rời xa tôi vì tôi lỡ yêu người mất rồi !
 

Welcome to My Facebook !


#279
Zaraki

Zaraki

    PQT

  • Phó Quản lý Toán Cao cấp
  • 4273 Bài viết


Ủng hộ topic một số bài trong Junior Balkan Mathematical Olympiad nào :))

Bài 155 : Tìm các số nguyên dương $x$ sao cho $n^{2}+3^{n}$ là số chính phương.

(Junior Balkan 2000)

Lời giải. Đặt $n^2+3^n=a^2$ với $a \in \mathbb{N}^*$ và $n=3^p \cdot q$ với $p,q \in \mathbb{N}, \; \gcd (3,q)=1$.

Vì $n \ge 1$ nên $q \ge 1$. Do đó $3^p \cdot q > 2p$.

Ta suy ra $3^{2p} |n^2+3^n$. Do đó $3^{p} |a$. Đặt $a=3^p \cdot k$ với $k \in \mathbb{N}^*, \; \gcd (k,3)=1$.

Khi đó phương trình trở thành $q^2+3^{h}=k^2$ với $h=3^p \cdot q-2p \in \mathbb{N}^*, \; \gcd (3,q)=1, \; \gcd (k,3)=1$.

Khi đó phương trình tương đương với $$\left( k-q \right) \left( k+q \right) = 3^h$$

Đặt $k-q=3^a, \; k+q=3^b$ với $a,b \in \mathbb{N} , \; a<b, \; a+b=h$. Khi đó $2q=3^a \left( 3^{b-a}-1 \right)$.

Vì $\gcd (q,3)=1$ nên $a=0$. Do đó $2q=3^h-1$ hay $2q= 3^{3^p \cdot q-2p}-1 \qquad (1)$.

 

Ta sẽ chứng minh bằng quy nạp hai bổ đề sau:

 

Bổ đề 1. Với mọi $p \in \mathbb{N}$ thì $3^p \ge 2p+1$. 

Chứng minh

Bổ đề 2. Với mọi $q \in \mathbb{N}^*$ thì ta luôn có $3^p \cdot q \ge 2p+q$ (trong đó $p \in \mathbb{N}$).

Chứng minh.

 

Quay lại $(1)$. Theo bổ để $2$ ta có $3^p \cdot q-2p \ge q \Rightarrow 2q+1 \ge 3^q-1$.

Tuy nhiên theo bổ đề $1$ nên ta suy ra $q=1$. Khi đó $3^p=2p+1$.

Áp dụng một lần nữa bổ để $1$ thì dẫn đến $p=0$ hoặc $p=1$. Vậy $n=1$ hoặc $n=3$.

Vậy số tự nhiên $n$ cần tìm là $\boxed{ n \in \{ 1;3 \}}$.


Discovery is a child’s privilege. I mean the small child, the child who is not afraid to be wrong, to look silly, to not be serious, and to act differently from everyone else. He is also not afraid that the things he is interested in are in bad taste or turn out to be different from his expectations, from what they should be, or rather he is not afraid of what they actually are. He ignores the silent and flawless consensus that is part of the air we breathe – the consensus of all the people who are, or are reputed to be, reasonable.

 

Grothendieck, Récoltes et Semailles (“Crops and Seeds”). 


#280
DarkBlood

DarkBlood

    Thiếu úy

  • Thành viên
  • 619 Bài viết

Bài 158 : Tìm nghiệm nguyên tố $p,q,r$ của phương trình sau :

$\frac{p}{q}-\frac{4}{r+1}=1$

(Junior Balkan 2008)

Ta có: $\frac{p}{q}-\frac{4}{r+1}=1\ \Leftrightarrow \ p(r+1)-4q=q(r+1)\ \Leftrightarrow \ (p-q)(r+1)=4q$

Vì $r+1\geq 3$ $($do $r$ là số nguyên tố$)$ và $4q>0$ nên $p-q>0$ hay $p>q$ mà $p$ và $q$ là số nguyên tố nên $p$ lẻ.

Vì $q$ là số nguyên tố, $r+1\geq 3,$ $p-q>0$ nên có bốn trường hợp có thể xảy ra:

 

$\bullet$ Trường hợp 1: $\left\{\begin{matrix} r+1=4q\\ p-q=1 \end{matrix}\right.$

Vì $p$ lẻ và $p-q=1$ nên $q=2$ $($do $q$ là số nguyên tố$),$ khi đó $p=3$ và $r=7,$ là số nguyên tố.

 

$\bullet$ Trường hợp 2: $\left\{\begin{matrix} r+1=4\\ p-q=q \end{matrix}\right.$

Vì $p-q=q$ nên $p=2q,$ do đó $p$ chẵn $($vô lý$).$

 

$\bullet$ Trường hợp 3: $\left\{\begin{matrix} r+1=q\\ p-q=4 \end{matrix}\right.$

Vì $p$ lẻ và $p-q=4$ nên $q$ lẻ, mà $r+1=q$ nên $r$ chẵn. Do đó $r=2$ $($Vì $r$ là số nguyên tố$),$ khi đó $q=3$ và $p=7,$ là số nguyên tố.

 

$\bullet$ Trường hợp 4: $\left\{\begin{matrix} r+1=2q\\ p-q=2 \end{matrix}\right.$

Vì $p$ lẻ và $p-q=2$ nên $q$ lẻ. Do đó $q\geq 3.$

Nếu $q=3$ thì $p=r=5.$

Nếu $q>3$ thì $q\equiv 1, 2\ (\bmod\ 3).$

     Xét $q\equiv 1\ (\bmod\ 3).$

Vì $p>q$ nên $p>3,$ do đó $p$ không chia hết cho $3.$

Ta có: $p-q=2\ \Leftrightarrow\ p=2+q\equiv 2+1\equiv 0\ (\bmod\ 3)$ $($Vô lý).$

     Xét $q\equiv 2\ (\bmod\ 3).$

Vì $r+1=2q$ và $q>3$ nên $r>3,$ do đó $r$ không chia hết cho $3.$

Ta có: $r+1=2q\ \Leftrightarrow\ r=2q-1\equiv 2.2-1\equiv 0\ (\bmod\ 3)$ $($Vô lý$).$

 

Vậy phương trình có nghiệm $$(p\ ;\ q\ ;\ r)=(3\ ;\ 2\ ;\ 7)\ ;\ (7\ ;\ 3\ ;\ 2)\ ;\ (5\ ;\ 3\ ;\ 5)$$






1 người đang xem chủ đề

0 thành viên, 1 khách, 0 thành viên ẩn danh